Zeros of partial sums of the exponential [duplicate]











up vote
4
down vote

favorite
1













This question already has an answer here:




  • Complex zeros of the polynomials $sum_{k=0}^{n} z^k/k!$, inside balls

    3 answers



  • How prove this $|z|>1$ with $1+z+frac{z^2}{2!}+cdots+frac{z^n}{n!}=0$

    1 answer




I am trying to show that if $$f_n(z)=1+z+frac{z^2}{2!}+...+frac{z^n}{n!}$$
Then $f_n(z)$ don’t have zeros inside the unitary disk.
I have tryied to use Rouche’s theorem or use that in the limit the polinomial converges to the exponential, but i dont get hoy to do this.










share|cite|improve this question













marked as duplicate by Martin R, Brahadeesh, Lord Shark the Unknown, Kelvin Lois, LutzL Nov 18 at 19:58


This question has been asked before and already has an answer. If those answers do not fully address your question, please ask a new question.















  • It converges locally uniformly to an entire function $f$. So if $f$ has no zeros it is finished. Otherwise assume $f$ has a zero of order $k$ at $z=a$, $f(z) sim C (z-a)^k$. Fix $epsilon$ very small, there is $N$ such that every $f_n,n ge N$ have $k$ zeros on $|z-a| < epsilon$ and $f_n(z) = h_n(z)prod_{l=1}^k (z-a_{l,n})=(C+O(z-a))prod_{l=1}^k (z-a_{l,n})$, $frac{f_n'}{f_n}(z) = O(z-a) + sum_{l=1}^k frac{1}{z-a_{l,n}}$. But $frac{f_n'}{f_n}(z) = 1-frac{frac{z^n}{n!}}{f_n(z)}=1-frac{a^n+O(z-a)}{n! (C+O(z-a))} frac{1}{prod_{l=1}^k (z-a_{l,n})}$, a contradiction.
    – reuns
    Nov 18 at 4:45












  • @reuns: Unless I am mistaken, your argument shows only that $f_n$ has no zeros in the unit disk for sufficiently large $n$.
    – Martin R
    Nov 18 at 8:31















up vote
4
down vote

favorite
1













This question already has an answer here:




  • Complex zeros of the polynomials $sum_{k=0}^{n} z^k/k!$, inside balls

    3 answers



  • How prove this $|z|>1$ with $1+z+frac{z^2}{2!}+cdots+frac{z^n}{n!}=0$

    1 answer




I am trying to show that if $$f_n(z)=1+z+frac{z^2}{2!}+...+frac{z^n}{n!}$$
Then $f_n(z)$ don’t have zeros inside the unitary disk.
I have tryied to use Rouche’s theorem or use that in the limit the polinomial converges to the exponential, but i dont get hoy to do this.










share|cite|improve this question













marked as duplicate by Martin R, Brahadeesh, Lord Shark the Unknown, Kelvin Lois, LutzL Nov 18 at 19:58


This question has been asked before and already has an answer. If those answers do not fully address your question, please ask a new question.















  • It converges locally uniformly to an entire function $f$. So if $f$ has no zeros it is finished. Otherwise assume $f$ has a zero of order $k$ at $z=a$, $f(z) sim C (z-a)^k$. Fix $epsilon$ very small, there is $N$ such that every $f_n,n ge N$ have $k$ zeros on $|z-a| < epsilon$ and $f_n(z) = h_n(z)prod_{l=1}^k (z-a_{l,n})=(C+O(z-a))prod_{l=1}^k (z-a_{l,n})$, $frac{f_n'}{f_n}(z) = O(z-a) + sum_{l=1}^k frac{1}{z-a_{l,n}}$. But $frac{f_n'}{f_n}(z) = 1-frac{frac{z^n}{n!}}{f_n(z)}=1-frac{a^n+O(z-a)}{n! (C+O(z-a))} frac{1}{prod_{l=1}^k (z-a_{l,n})}$, a contradiction.
    – reuns
    Nov 18 at 4:45












  • @reuns: Unless I am mistaken, your argument shows only that $f_n$ has no zeros in the unit disk for sufficiently large $n$.
    – Martin R
    Nov 18 at 8:31













up vote
4
down vote

favorite
1









up vote
4
down vote

favorite
1






1






This question already has an answer here:




  • Complex zeros of the polynomials $sum_{k=0}^{n} z^k/k!$, inside balls

    3 answers



  • How prove this $|z|>1$ with $1+z+frac{z^2}{2!}+cdots+frac{z^n}{n!}=0$

    1 answer




I am trying to show that if $$f_n(z)=1+z+frac{z^2}{2!}+...+frac{z^n}{n!}$$
Then $f_n(z)$ don’t have zeros inside the unitary disk.
I have tryied to use Rouche’s theorem or use that in the limit the polinomial converges to the exponential, but i dont get hoy to do this.










share|cite|improve this question














This question already has an answer here:




  • Complex zeros of the polynomials $sum_{k=0}^{n} z^k/k!$, inside balls

    3 answers



  • How prove this $|z|>1$ with $1+z+frac{z^2}{2!}+cdots+frac{z^n}{n!}=0$

    1 answer




I am trying to show that if $$f_n(z)=1+z+frac{z^2}{2!}+...+frac{z^n}{n!}$$
Then $f_n(z)$ don’t have zeros inside the unitary disk.
I have tryied to use Rouche’s theorem or use that in the limit the polinomial converges to the exponential, but i dont get hoy to do this.





This question already has an answer here:




  • Complex zeros of the polynomials $sum_{k=0}^{n} z^k/k!$, inside balls

    3 answers



  • How prove this $|z|>1$ with $1+z+frac{z^2}{2!}+cdots+frac{z^n}{n!}=0$

    1 answer








complex-analysis power-series taylor-expansion






share|cite|improve this question













share|cite|improve this question











share|cite|improve this question




share|cite|improve this question










asked Nov 18 at 3:25









J.Rodriguez

15310




15310




marked as duplicate by Martin R, Brahadeesh, Lord Shark the Unknown, Kelvin Lois, LutzL Nov 18 at 19:58


This question has been asked before and already has an answer. If those answers do not fully address your question, please ask a new question.






marked as duplicate by Martin R, Brahadeesh, Lord Shark the Unknown, Kelvin Lois, LutzL Nov 18 at 19:58


This question has been asked before and already has an answer. If those answers do not fully address your question, please ask a new question.














  • It converges locally uniformly to an entire function $f$. So if $f$ has no zeros it is finished. Otherwise assume $f$ has a zero of order $k$ at $z=a$, $f(z) sim C (z-a)^k$. Fix $epsilon$ very small, there is $N$ such that every $f_n,n ge N$ have $k$ zeros on $|z-a| < epsilon$ and $f_n(z) = h_n(z)prod_{l=1}^k (z-a_{l,n})=(C+O(z-a))prod_{l=1}^k (z-a_{l,n})$, $frac{f_n'}{f_n}(z) = O(z-a) + sum_{l=1}^k frac{1}{z-a_{l,n}}$. But $frac{f_n'}{f_n}(z) = 1-frac{frac{z^n}{n!}}{f_n(z)}=1-frac{a^n+O(z-a)}{n! (C+O(z-a))} frac{1}{prod_{l=1}^k (z-a_{l,n})}$, a contradiction.
    – reuns
    Nov 18 at 4:45












  • @reuns: Unless I am mistaken, your argument shows only that $f_n$ has no zeros in the unit disk for sufficiently large $n$.
    – Martin R
    Nov 18 at 8:31


















  • It converges locally uniformly to an entire function $f$. So if $f$ has no zeros it is finished. Otherwise assume $f$ has a zero of order $k$ at $z=a$, $f(z) sim C (z-a)^k$. Fix $epsilon$ very small, there is $N$ such that every $f_n,n ge N$ have $k$ zeros on $|z-a| < epsilon$ and $f_n(z) = h_n(z)prod_{l=1}^k (z-a_{l,n})=(C+O(z-a))prod_{l=1}^k (z-a_{l,n})$, $frac{f_n'}{f_n}(z) = O(z-a) + sum_{l=1}^k frac{1}{z-a_{l,n}}$. But $frac{f_n'}{f_n}(z) = 1-frac{frac{z^n}{n!}}{f_n(z)}=1-frac{a^n+O(z-a)}{n! (C+O(z-a))} frac{1}{prod_{l=1}^k (z-a_{l,n})}$, a contradiction.
    – reuns
    Nov 18 at 4:45












  • @reuns: Unless I am mistaken, your argument shows only that $f_n$ has no zeros in the unit disk for sufficiently large $n$.
    – Martin R
    Nov 18 at 8:31
















It converges locally uniformly to an entire function $f$. So if $f$ has no zeros it is finished. Otherwise assume $f$ has a zero of order $k$ at $z=a$, $f(z) sim C (z-a)^k$. Fix $epsilon$ very small, there is $N$ such that every $f_n,n ge N$ have $k$ zeros on $|z-a| < epsilon$ and $f_n(z) = h_n(z)prod_{l=1}^k (z-a_{l,n})=(C+O(z-a))prod_{l=1}^k (z-a_{l,n})$, $frac{f_n'}{f_n}(z) = O(z-a) + sum_{l=1}^k frac{1}{z-a_{l,n}}$. But $frac{f_n'}{f_n}(z) = 1-frac{frac{z^n}{n!}}{f_n(z)}=1-frac{a^n+O(z-a)}{n! (C+O(z-a))} frac{1}{prod_{l=1}^k (z-a_{l,n})}$, a contradiction.
– reuns
Nov 18 at 4:45






It converges locally uniformly to an entire function $f$. So if $f$ has no zeros it is finished. Otherwise assume $f$ has a zero of order $k$ at $z=a$, $f(z) sim C (z-a)^k$. Fix $epsilon$ very small, there is $N$ such that every $f_n,n ge N$ have $k$ zeros on $|z-a| < epsilon$ and $f_n(z) = h_n(z)prod_{l=1}^k (z-a_{l,n})=(C+O(z-a))prod_{l=1}^k (z-a_{l,n})$, $frac{f_n'}{f_n}(z) = O(z-a) + sum_{l=1}^k frac{1}{z-a_{l,n}}$. But $frac{f_n'}{f_n}(z) = 1-frac{frac{z^n}{n!}}{f_n(z)}=1-frac{a^n+O(z-a)}{n! (C+O(z-a))} frac{1}{prod_{l=1}^k (z-a_{l,n})}$, a contradiction.
– reuns
Nov 18 at 4:45














@reuns: Unless I am mistaken, your argument shows only that $f_n$ has no zeros in the unit disk for sufficiently large $n$.
– Martin R
Nov 18 at 8:31




@reuns: Unless I am mistaken, your argument shows only that $f_n$ has no zeros in the unit disk for sufficiently large $n$.
– Martin R
Nov 18 at 8:31










1 Answer
1






active

oldest

votes

















up vote
0
down vote













You have all the facts you need. Here is an outline of what you need to do:



Use Rouche's theorem with $f_n$ and $z^nover n!$ to show that if some $f_N$ has a zero within the unit disk, then every $f_n$ with $ngt N$ has a zero in the unit disk. Now since there are an infinite number of zeros in the unit disk, the set of points mapping to zero under our $f_n$'s has at least one accumulation point. So we have a sequence of pairs, $f_i, p_i$ with $f_i(p_i)=0$ for each i, with a limit point $f, p$ (as the $f_n$'s also converge). For the sake of contradiction, we want to show $f(p)=0$ as we already know that f is the exponential and so has no zeros.



To do this use an epsilon delta argument: let $epsilon gt 0$, then find $N_0$ such that $f_n(x)$ is within $epsilon over 3$ of f(x) for all x in the unit disk when $ngt N_0$, then find $delta$ such that when $|x-x_0|ltdelta$, then $|f(x)-f(x_0)| lt$ $ epsilon over 3$. Lastly find $N_1$ so that $ngt N_1$ implies $|p-p_n| gt delta$. Use the triangle inequality to show that when $n>max(N_0,N_1)$ we have $|f(p)-f_n(p_n)|<epsilon$. This completes the contradiction, as it implies $f(x)=e^x$ has a zero in the unit disk.






share|cite|improve this answer




























    1 Answer
    1






    active

    oldest

    votes








    1 Answer
    1






    active

    oldest

    votes









    active

    oldest

    votes






    active

    oldest

    votes








    up vote
    0
    down vote













    You have all the facts you need. Here is an outline of what you need to do:



    Use Rouche's theorem with $f_n$ and $z^nover n!$ to show that if some $f_N$ has a zero within the unit disk, then every $f_n$ with $ngt N$ has a zero in the unit disk. Now since there are an infinite number of zeros in the unit disk, the set of points mapping to zero under our $f_n$'s has at least one accumulation point. So we have a sequence of pairs, $f_i, p_i$ with $f_i(p_i)=0$ for each i, with a limit point $f, p$ (as the $f_n$'s also converge). For the sake of contradiction, we want to show $f(p)=0$ as we already know that f is the exponential and so has no zeros.



    To do this use an epsilon delta argument: let $epsilon gt 0$, then find $N_0$ such that $f_n(x)$ is within $epsilon over 3$ of f(x) for all x in the unit disk when $ngt N_0$, then find $delta$ such that when $|x-x_0|ltdelta$, then $|f(x)-f(x_0)| lt$ $ epsilon over 3$. Lastly find $N_1$ so that $ngt N_1$ implies $|p-p_n| gt delta$. Use the triangle inequality to show that when $n>max(N_0,N_1)$ we have $|f(p)-f_n(p_n)|<epsilon$. This completes the contradiction, as it implies $f(x)=e^x$ has a zero in the unit disk.






    share|cite|improve this answer

























      up vote
      0
      down vote













      You have all the facts you need. Here is an outline of what you need to do:



      Use Rouche's theorem with $f_n$ and $z^nover n!$ to show that if some $f_N$ has a zero within the unit disk, then every $f_n$ with $ngt N$ has a zero in the unit disk. Now since there are an infinite number of zeros in the unit disk, the set of points mapping to zero under our $f_n$'s has at least one accumulation point. So we have a sequence of pairs, $f_i, p_i$ with $f_i(p_i)=0$ for each i, with a limit point $f, p$ (as the $f_n$'s also converge). For the sake of contradiction, we want to show $f(p)=0$ as we already know that f is the exponential and so has no zeros.



      To do this use an epsilon delta argument: let $epsilon gt 0$, then find $N_0$ such that $f_n(x)$ is within $epsilon over 3$ of f(x) for all x in the unit disk when $ngt N_0$, then find $delta$ such that when $|x-x_0|ltdelta$, then $|f(x)-f(x_0)| lt$ $ epsilon over 3$. Lastly find $N_1$ so that $ngt N_1$ implies $|p-p_n| gt delta$. Use the triangle inequality to show that when $n>max(N_0,N_1)$ we have $|f(p)-f_n(p_n)|<epsilon$. This completes the contradiction, as it implies $f(x)=e^x$ has a zero in the unit disk.






      share|cite|improve this answer























        up vote
        0
        down vote










        up vote
        0
        down vote









        You have all the facts you need. Here is an outline of what you need to do:



        Use Rouche's theorem with $f_n$ and $z^nover n!$ to show that if some $f_N$ has a zero within the unit disk, then every $f_n$ with $ngt N$ has a zero in the unit disk. Now since there are an infinite number of zeros in the unit disk, the set of points mapping to zero under our $f_n$'s has at least one accumulation point. So we have a sequence of pairs, $f_i, p_i$ with $f_i(p_i)=0$ for each i, with a limit point $f, p$ (as the $f_n$'s also converge). For the sake of contradiction, we want to show $f(p)=0$ as we already know that f is the exponential and so has no zeros.



        To do this use an epsilon delta argument: let $epsilon gt 0$, then find $N_0$ such that $f_n(x)$ is within $epsilon over 3$ of f(x) for all x in the unit disk when $ngt N_0$, then find $delta$ such that when $|x-x_0|ltdelta$, then $|f(x)-f(x_0)| lt$ $ epsilon over 3$. Lastly find $N_1$ so that $ngt N_1$ implies $|p-p_n| gt delta$. Use the triangle inequality to show that when $n>max(N_0,N_1)$ we have $|f(p)-f_n(p_n)|<epsilon$. This completes the contradiction, as it implies $f(x)=e^x$ has a zero in the unit disk.






        share|cite|improve this answer












        You have all the facts you need. Here is an outline of what you need to do:



        Use Rouche's theorem with $f_n$ and $z^nover n!$ to show that if some $f_N$ has a zero within the unit disk, then every $f_n$ with $ngt N$ has a zero in the unit disk. Now since there are an infinite number of zeros in the unit disk, the set of points mapping to zero under our $f_n$'s has at least one accumulation point. So we have a sequence of pairs, $f_i, p_i$ with $f_i(p_i)=0$ for each i, with a limit point $f, p$ (as the $f_n$'s also converge). For the sake of contradiction, we want to show $f(p)=0$ as we already know that f is the exponential and so has no zeros.



        To do this use an epsilon delta argument: let $epsilon gt 0$, then find $N_0$ such that $f_n(x)$ is within $epsilon over 3$ of f(x) for all x in the unit disk when $ngt N_0$, then find $delta$ such that when $|x-x_0|ltdelta$, then $|f(x)-f(x_0)| lt$ $ epsilon over 3$. Lastly find $N_1$ so that $ngt N_1$ implies $|p-p_n| gt delta$. Use the triangle inequality to show that when $n>max(N_0,N_1)$ we have $|f(p)-f_n(p_n)|<epsilon$. This completes the contradiction, as it implies $f(x)=e^x$ has a zero in the unit disk.







        share|cite|improve this answer












        share|cite|improve this answer



        share|cite|improve this answer










        answered Nov 18 at 5:07









        Mark

        3916




        3916















            Popular posts from this blog

            Biblatex bibliography style without URLs when DOI exists (in Overleaf with Zotero bibliography)

            ComboBox Display Member on multiple fields

            Is it possible to collect Nectar points via Trainline?